Đến nội dung

Hình ảnh

Chứng minh rằng giữa các hiệu $a_{i}$ - $a_{k}$ ( $a_{i}$ > $a_{k}$ ) luôn tìm đươc ít nhất 4 hiệu bằng nhau.


  • Please log in to reply
Chủ đề này có 2 trả lời

#1
Math Is Love

Math Is Love

    $\mathfrak{Forever}\ \mathfrak{Love}$

  • Thành viên
  • 620 Bài viết
Bài 1: Chứng minh rằng nếu a và b là các số nguyên dương có thể biểu diễn dưới dạng: $\dfrac{a}{b}$ =1+ $\dfrac{1}{2}$ + $\dfrac{1}{3}$ +......+ $\dfrac{1}{p-1}$ thì a $\vdots$ p trong đó p là số nguyên tố lẻ.
Bài 2: Cho 20 số tự nhiên khác nhau $a_{1}$ , $a_{2}$ , $a_{3}$ ,........, $a_{20}$ không vượt quá 70. Chứng minh rằng giữa các hiệu $a_{i}$ - $a_{k}$ ( $a_{i}$ > $a_{k}$ ) luôn tìm đươc ít nhất 4 hiệu bằng nhau.

Bài viết đã được chỉnh sửa nội dung bởi L Lawliet: 14-07-2012 - 21:04

Hình đã gửi


#2
defaw

defaw

    Hạ sĩ

  • Thành viên
  • 52 Bài viết
Bài 1:Giải:
Ta có: $\frac{a}{b}=(1+\frac{1}{p-1})+(\frac{1}{2}+\frac{1}{p-2})+...+(\frac{1}{\frac{p-1}{2}}+\frac{1}{\frac{p+1}{2}})$
$=\frac{p}{p-1}+\frac{p}{2(p-2)}+...\frac{p}{\frac{(p-1)(p+1)}{4}}=p\cdot \frac{m}{n}$
Suy ra $a\cdot n = b\cdot pm$, mà $n = (p-1)\cdot 2(p-2)...\frac{(p-1)(p+1)}{4}$nguyên tố cùng nhau với $p$.
$\cdot$ Do đó $a\vdots p$.

#3
Strygwyr

Strygwyr

    Sk8er-boi

  • Thành viên
  • 272 Bài viết

Bài 2: Cho 20 số tự nhiên khác nhau $a_{1}$ , $a_{2}$ , $a_{3}$ ,........, $a_{20}$ không vượt quá 70. Chứng minh rằng giữa các hiệu $a_{i}$ - $a_{k}$ ( $a_{i}$ > $a_{k}$ ) luôn tìm đươc ít nhất 4 hiệu bằng nhau.

Còn bài $2$ nữa mọi người nè :P

Giả sử kết luận của bài toán không đúng.Không mất tính tổng quát, giả sử $a_{20}> a_{19}>a_{18}>...>a_1$.Khi đó xét riêng $19$ hiệu sau : 

$a_{20}-a_{19}$,$a_{19}-a_{18}$,...,$a_3-a_2$,$a_2-a_1$

không có bốn số nào bằng nhau. Do đó lại nói riêng trong dãy $19$ số đó, mỗi số $1$,$2$,$3$,$4$,$5$,$6$ có mặt không quá $3$ lần. Từ đó, trong $19$ số này phải có ít nhất $1$ số lớn hơn $6$. Một cách tương tự thì trong $18$ số có ít nhất $3$ số lớn hơn $5$, trong $15$ số còn lại có ít nhất $3$ số lớn hơn $4$,... Vì thế :

($a_{20}-a_{19}$)$+$($a_{19}-a_{18}$)$+$...$+$($a_2-a_1$)$\geq 7+3(6+5+4+3+2+1)=70$

Do đó, $a_{20}-a_1\geq 70$.

Mặt khác : $1\leq a_i\leq 70$ với mọi $i=$$1$,$2$,...,$20$  suy ra $a_{20}-a_1\leq 69$ (mâu thuẫn)

Vậy giả thiết phản chứng sai nên ta được đpcm.


Bài viết đã được chỉnh sửa nội dung bởi namsub: 07-07-2013 - 11:07

"Nothing is impossible"

(Napoleon Bonaparte)





1 người đang xem chủ đề

0 thành viên, 1 khách, 0 thành viên ẩn danh